1968 AHSME Problems/Problem 29

Revision as of 21:42, 17 July 2024 by Thepowerful456 (talk | contribs) (Solution)

Problem

Given the three numbers $x,y=x^x,z=x^{x^x}$ with $.9<x<1.0$. Arranged in order of increasing magnitude, they are:

$\text{(A) } x,z,y\quad \text{(B) } x,y,z\quad \text{(C) } y,x,z\quad \text{(D) } y,z,x\quad \text{(E) } z,x,y$

Solution

Seeing that we need to compare values with exponents, we think logarithms. Taking the logarithm base $x$ of each term, we obtain $1$, $x$, and $x^x$. Because $0<x<1$, $f(n)=\log_x(n)$ is monotonically decreasing, so the order of terms by magnitude in our new set of numbers will be reversed compared to the original set (i.e. if $a<b<c$, then $\log_x(a)>\log_x(b)>\log_x(c))$. However, the order of this set will be reversed again (back to the order of the original set) when we take the logarithm base $x$ a second time. After doing this operation, we find the values $0$, $1$, and $x$, which correspond to $x$, $y$, and $z$, respectively. Because $0.9<x<1$, $0<x<1$, and so, by the correspondence detailed above, $x<z<y$, which yields us answer choice $\fbox{A}$.

See also

1968 AHSC (ProblemsAnswer KeyResources)
Preceded by
Problem 28
Followed by
Problem 30
1 2 3 4 5 6 7 8 9 10 11 12 13 14 15 16 17 18 19 20 21 22 23 24 25 26 27 28 29 30 31 32 33 34 35
All AHSME Problems and Solutions

The problems on this page are copyrighted by the Mathematical Association of America's American Mathematics Competitions. AMC logo.png